RegistrierenRegistrieren   LoginLogin   FAQFAQ    SuchenSuchen   
Frage zur Quantenschleifengravitationstheorie
 
Neue Frage »
Antworten »
    Foren-Übersicht -> Quantenphysik
Autor Nachricht
Kassiopeija



Anmeldungsdatum: 26.06.2015
Beiträge: 146

Beitrag Kassiopeija Verfasst am: 22. Nov 2015 07:42    Titel: Frage zur Quantenschleifengravitationstheorie Antworten mit Zitat

Vorab mal sorry, für Physiker wird sich das jetzt gräßlich lesen...

Ich hab vor einiger Zeit mal auf en.wikpedia über obige Theorie gelesen, und da stand was dabei daß man versucht, die Quantelung des Raums zu beweisen indem man Lichtwellen untersucht, die mrd Jahre unterwegs waren, und denen irgendwelche Information anscheinend abhanden kommen würde.

In dem Artikel (den ich dummerweise nicht mehr find...) stand auch daß man diese Messungen mittels Satellit gemacht hätte, aber es sich nicht bestätigt hätte. Aber bei Nachrechnungen wurde eingewndet daß es Meßfehler ua Rechenfehler gegeben haben soll.... was der Artikel allerdings verschwieg ist ob die erneute + korrekte Nachrechnung bzw. Messung dann die erwarteten Ergebnisse geliefert hat. Weiß dazu jemand was?

Gruß
TomS
Moderator


Anmeldungsdatum: 20.03.2009
Beiträge: 18062

Beitrag TomS Verfasst am: 22. Nov 2015 10:19    Titel: Antworten mit Zitat

Das ist ein noch nicht vollständig verstandenes Problemfeld.

Zunächst mal muss man unterscheiden zwischen "naiven Argumenten" und spezifischen Theorien.

Naive Argumente funktionieren so: "Quantelung des Raumes resultieren in einer minimalen Länge. Eine minimale Länge bricht die kontinuierliche Translationsinvarianz. Daraus folgt, dass der Zusammenhang zwischen Energie, Impuls und Geschwindigkeit modifiziert wird. Dies führt zu einer frequenzabhängigen Lichtgeschwindigkeit".

Man kann sogenannte modifizierte Dispersionsrelationen, die letztlich eine Deformation oder Brechung der lokalen Lorentzinvarianz bedeuten, allgemein hinschreiben und versuchen, die darin auftretenden Parameter experimentell zu bestimmen. Im Rahmen der heute erzielbaren Messgenauigkeit existieren keine Hinweise auf eine derartige Deformation.

Eine spezifische Theorie ist nun die LQG. Für diese wurde vor m.W.n. ca. 10 Jahren eine - wie man inzwischen weiß unzulässige - Näherung entwickelt, aus der eine derartige Defirmation folgen sollte. Das ist inzwischen vom Tisch, wird jedoch immer wieder als Argument gegen die LQG angeführt, obwohl diese in ihrer modernen Fassung keine diesbzgl. Aussagen macht, und obwohl die Messgenauigkeit ohnehin nicht ausreicht, um bestimmte Ansötze auszuschließen, man kann sie lediglich einschränken.

Laut Rovelli liegt eine manifest lokal-Lorentzinvariante Formulierung vor:

Zitat:
The answer is roughly that LQG does not in fact violate Lorentz invariance. The discretisation of area and volume operators does not imply a broken symmetry, any more than discretisation of angular momentum states imply breaking of rotational symmetry --- symmetries in quantum theories are equations of the operator algebra, not of the states!

See also: http://arxiv.org/abs/1012.173

Carlo Rovelli hat Folgendes geschrieben:
This is the correct answer. LQG is Lorentz invariant. To see details, see the two papers fr.arxiv.org/abs/1012.1739 (recent, shows the Lorentz covanraince of LGG explicitly) and fr.arxiv.org/abs/gr-qc/0205108 (explains in detail why the argument about shrinking of the minimal area is wrong. that is, gives the details behind the point made by gennetg.) – Carlo Rovelli Jan 28 '11 at 14:32


Es gibt jedoch m.E. immer noch einige berechtigte Zweifel, ob die sogenannte Operator-Algebra der LQG vollständig konsistent (anomalienfrei) ist; außerdem gibt es in dem Zusammenhang m.E. gewisse Freiheiten, wie die Theorie konstruiert werden kann, d.h. evtl. muss man eher von einer Klasse von Theorien sprechen. Rovelli bezieht sich auf ein speziell von ihm untersuchte Variante.

Zusammenfassend würde ich sagen, dass das Thema noch nicht endgültig geklärt ist, dass jedoch die o.g. naiven Argumente ins Leere laufen.
Kassiopeija



Anmeldungsdatum: 26.06.2015
Beiträge: 146

Beitrag Kassiopeija Verfasst am: 23. Nov 2015 07:35    Titel: Antworten mit Zitat

Ok danke TomS für die Antwort.
Neue Frage »
Antworten »
    Foren-Übersicht -> Quantenphysik